Which expressions are equivalent to 10/10^3/4?

Which Expressions Are Equivalent To 10/10^3/4?

Answers

Answer 1

[tex] \frac{10}{10 {}^{ \frac{3}{4} } } = 10 {}^{1 - \frac{3}{4} } = 10 { }^{ \frac{4 - 3}{4} } [/tex]

[tex]10 {}^{ \frac{ 1}{4} } [/tex]

Answer 2

Answer: B and D

Step-by-step explanation:

Edge 2023


Related Questions

Two students determined the value of this expression.

-2.5(1.4+3.1) + 6.9(-4.3)

These are the steps each student used.



Student P

Step 1: -3.5 + 7.75 + 6.9(-4.3)



Step 2: -3.5 + 7.75 - 29.67



Step 3: 7.75 - 3.5 - 29.6
Step 4: -25.42

Student Q
Step 1: -3.5 - 7.75 + 6.9(-4.3)
Step 2: -3.5 - 7.75 - 29.67
Step 3: -(3.5 - 7.75 - 29.67)
Step 4: -(-33.92)
Step 5: 33.92
Analyze the steps and describe any errors made by Student P and Student Q.

Answers

Let's solve

[tex]\\ \rm\rightarrowtail -2.5(1.4+3.1)+6.9(-4.3)[/tex]

[tex]\\ \rm\rightarrowtail -3.5-7.75-29.67[/tex]

[tex]\\ \rm\rightarrowtail -11.25-29.67[/tex]

[tex]\\ \rm\rightarrowtail -40.92[/tex]

Both did error in signs

Answer:

Given expression:

-2.5(1.4+3.1) + 6.9(-4.3)

Student P

Step 1:  -3.5 + 7.75 + 6.9(-4.3)

Step 2: -3.5 + 7.75 - 29.67

Step 3: 7.75 - 3.5 - 29.67

Step 4: -25.42

Student P made an error in step 1.  When expanding the first part of the expression, they forgot the negative sign when multiplying -2.5 by 3.1.  

So Step 1 should be:  -3.5 - 7.75 + 6.9(-4.3).  

This error then impacted all following steps and gave an incorrect final solution.

Student Q

Step 1: -3.5 - 7.75 + 6.9(-4.3)

Step 2: -3.5 - 7.75 - 29.67

Step 3: -(3.5 - 7.75 - 29.67)

Step 4: -(-33.92)

Step 5: 33.92

Student Q made an error in step 3.  When placing the numbers in parentheses and placing a negative sign outside the parentheses, they should have changed the two negative signs inside the parentheses to positives.

So Step 3 should be: -(3.5 + 7.75 + 29.67).  

This error then impacted all following steps and gave an incorrect final solution.

Correct steps and solution

Step 1:  -3.5 - 7.75 + 6.9(-4.3)

Step 2: -3.5 - 7.75 - 29.67

Step 3: -(3.5 + 7.75 + 29.67)

Step 4: -(40.92)

Step 5: -40.92

PLSSSS ANSWER THIS QUESTION FAST
WILL MARK BRAINLIEST

Answers

Answer:

150 minutes

Step-by-step explanation:

Total plants are 16 and only 5 can be placed in one go. so total number of rounds for the plants will be: 16/5 = 3.2 rounds

As there are 3.2 rounds to go in 8 hours, so the time for 1 round will be: 8/3.2= 2 hours and 30 minutes

Answer:

150 minutes

Step-by-step explanation:

16/5 = 3.2  turns

8 hours /3.2 turns  = 2.5 hours (150 minutes)

Help picture below problem 11

Answers

Answer:

128° is the correct answer...

Step-by-step explanation:

mark me brainliest

How to solve 4 divided by 5

Answers

Answer: HELLO THERE!  Here's what I found from study dot  com            "4 divided by 5 is equal to 0.8. This decimal can also be written as a fraction. 0.8 = eight tenths or 8/10 (4/5 in its reduced form)."

Step-by-step explanation:

Hope this helps, have a good day!

How to find an slope

Answers

Answer:

Y2-Y1

____

X2-X1

To FIND SLOPE. YOU DIVED THE DIFFERENCE OF THE Y- COORDINATES OF 2 POINTS ON LINE BY THE DIFFERENCE OF THE X- COORDINATES OF THOSE SAME 2 POINTS.

I THINK IT IS HELPFUL FOR YOU.?

show how you know, thank you :)
~giving brainliest!~

Answers

Answer:

V≈180.96

Step-by-step explanation:

Stacy performed an experiment in which she rolled a six-sided die multiple times. The results are shown below.
Based on the results above, predict the number of times a 3 would be rolled if Stacy performs the experiment a total of 480 times

Answers

Answer:

5/9

Step-by-step explanation:

Let the probability of getting a multiple of 3 is "A"

Let the probability of getting an even number is "B"

Total number of outcomes=36

Total number of favourable outcomes of either a multiple of 3 or an even number are

(1,1) ,(1,2) ,(2,1) ,(1,3) ,(3,1) ,(2,2) ,(1,5) (5,1) ,(3,3) ,(2,4) ,(4,2) ,(2,6) ,(6,2) ,(3,5) ,(5,3) ,(4,4) ,(6,4) ,(4,6) ,(5,5) ,(6,6)

That is we have 20 favourable outcomes

Hence, Probability=\frac{\text{favourable outcomes}}{\text{total number of outcomes}}Probability=

total number of outcomes

favourable outcomes

Therefore, after substituting the values we get

probability=\frac{20}{36}probability=

36

20

After simplification we will get

probability=\frac{5}{9}probability=

9

5

For each value of u, determine whether it is a solution to
- 57 = 5u - 7
2, 3, 10, 0

Answers

Answer:2 is a solution 3 is not 0 is not 10 is and also to determine the solution you can get calculator soup to help you Hope this helps bye

If you should feed 1½ lbs of hay per 100 lbs of your horse’s body weight, how much should you feed a 1100 lb horse?
A. 23lbs
B. 16.5lbs
C. 13.6lbs
D. 7.5lbs

Answers

I think it's 16.5lbs I'm sorry if I'm wrong I don't know for sure

Question Help
8.6.PS-13
A circular plate has circumference 28.3 inches. What is the area of this plate? Use 3.14 for it.
The area of this plate is about square inches.
(Round the final answer to the nearest whole number as needed. Round all intermediate values to the nearest
thousandth as needed.)

Answers

Answer:

Step-by-step explanation:

Area = 63.9 inches²

Help picture below problem 16

Answers

The Pythagorean theorem is the idea that the sum of the two legs which are both squared is equal to the hypotenuse's length squared.

     *look at the image I attached for a better explanation

By looking at the picture, we are missing the leg's length, and by using the Pythagorean theorem, we get the equation

    [tex]?^2+9^2 = 16^2\\?^2 + 81 = 256\\?^2 = 175\\? = 13.2[/tex]

Thus the missing side length is 13.2 cm

Hope that helps!

Jill leaves work from the High School, but has to pick up her friend at the airport before going back home. What is the shortest distance she can travel?

A. 27 miles
B. 21 miles
C. 56 miles
D. 84 miles

Answers

Answer:

  B.  21 miles

Step-by-step explanation:

The shortest overall route will be the sum of the shortest routes between way points.

The shortest route from the High School to the Airport is the direct route, 15 cm on the map. The shortest route from the Airport to Jill's house is via the High School and City Hall, a total of 15+12+21 = 48 cm on the map.

The shortest distance Jill can travel from work to home via the airport is ...

  (15 cm +48 cm)/(3 cm/mi) = 21 miles

Answer:

person above is correct it is 21

What is the value of g?

Answers

Answer:

90

Step-by-step explanation:

I'm not sure about the answer

g = 26

78 - 52 is 26 since 78 is the same as the other angle, we can conclude that they are equal to each other.

What is the result when you combine only the constants in the following expression?
3a-6b+3-7a+ 10

Answers

-4a, -6b + 13 would be the answer
The answer would be: -4a -6b +13
The breakdown:
Numbers in expression; 3a,-7a,-6b,10,3.

3a-7a = -4a
-6b = -6b
10+3 = 13
+++
=
- 4a - 6b + 13

Hope that helps you!

There are 3 times as many males as females on the maths course at university. What fraction of the course are male?
Give your answer in its simplest form.
Colin gets pic 'n mix when at the cinema and makes his bag up with 3 types of sweet. He picks 3 times as many cola bottles as smarties. He also picks twice as many smarties as marshmallows. What proportion of the bag of sweets are cola bottles?
Give your answer in its simplest form

Answers

(1)

If there are three times as many males as females, the ratio of males to females would be 3:1.

To make this more clearer. Suppose you have 100 students. for each 4 students, 3 of them are male and 1 is a female.

We can then say there are 75 males and 25 females.

Expressing this as a ratio(fraction) is easy 3 out of 4 are males and 1 out of 4 are females. So the ratio of males would be;

[tex]r(m) = \frac{3}{4} [/tex]

(2)

Let x be the number of cola bottles

Let y be the number of smarties

Let z be the number of marshmallows

x=3y (since there are 3 times more x than y)

y=2z (since there are 2 times more y than z)

we know that marshmallows are twice as less as smarties. so for 1 marshmallow you'd have 2 smarties. But we also know that cola bottles should be 2×3=6 since we have 3 times more cola bottles than smarties.

Adding up our total sweets we get;

1+2+6=9

But how many of those are cola bottles? 6!

So the ratio of cola bottles can be expressed as,

[tex]r(c) = \frac{6}{9} = \frac{2}{3} [/tex]

You can simplify the numerator and denominator by 3 to get 2 over 3.

Triangle ABC was dilated and translated to form similar triangle A'B'C'.

On a coordinate plane, 2 triangles are shown. Triangle A B C has points (0, 2), (2, 2), and (2, 0). Triangle A prime B prime C prime has points (negative 4, negative 1), (1, negative 1), and (1, negative 6).

What is the scale factor of the dilation?

One-fifth
Two-fifths
Five-halves
5

Answers

The scale factor of the dilation is Two-fifths.

Answer:

C

Step-by-step explanation:

Guys please help! i will mark brainliest to the right answers

Note: in order for me to mark a brainliest 2 people need to answer

Answers

Answer:

The answer is a (4,6) :)

0,4

2,5

4,6

Have an amazing day!!

Please rate and mark brainliest!!

Use point-slope form to solve for the slope.

(y-y,)=m(x-x,)
(4-5)=m(0-2)
m=1/2

we know the line passes through (0,4) so that is your y-intercept.

slope intercept form
y=mx+b
y=1/2 x + 4

now just plug in the points until you find one that is true:

Lucky for us the first one works (4,6)

funny that they give (2,5) as an option and it’s already in the question but they want you to find another point, so

The answer is A

I hope this helps and is the BRAINLIEST!

Good luck with your studies!

Solve sqrtx-11 + 7 = 19

Answers

Answer:

x = 155

Step-by-step explanation:

[tex]\sqrt{x-11} +7=19[/tex]

[tex]\sqrt{x-11} =12[/tex]

[tex](\sqrt{x-11} )^2=12^2[/tex]

[tex]x-11=144[/tex]

x = 155

please Show the algebraic work steps thank you

Answers

Answer:

-0.125

Step-by-step explanation:

Shown in the image above

Solve for x. Leave your answer in simplest radical form. X 4 3​

Answers

Apply Pythagorean theorem

P²=H²-B²P²=4²-2²P²=16-4P²=12

Now

Again apply Pythagorean theorem

x²=12-3²x²=12-9x²=3x=√3

Answer:

the final answer for this problem is √3

n/52 = 180/120 = n
18/n = 21/28 =n
n/4 = 24/6 =n
10/16 = n/56=n
solve each propotion

Answers

1. n/52 = 180/120
180/120 = 1.5
n/52 = 1.5
Multiply 52 on both sides to simplify:
n=78

2. 18/n = 21/28
21/28 = 0.75
18/n = 0.75
Multiply n on both sides to remove
fraction: 18 = 0.75n and divide both sides
by 0.75 to simplify again.
n=24

3. n/4 = 24/6
24/6 = 4
n/4 = 4
Multiply 4 by both sides to simplify:
n=16

4. n/56 = 10/16
10/16 = 0.625
n/56 = 0.625
Multiply 56 on both sides to simplify:
n=35


Hope this helps :))

Answer:

1. n/52 = 180/120

180/120 = 1.5

n/52 = 1.5

Multiply 52 on both sides to simplify:

n=78

2. 18/n = 21/28

21/28 = 0.75

18/n = 0.75

Multiply n on both sides to remove

fraction: 18 = 0.75n and divide both sides

by 0.75 to simplify again.

n=24

3. n/4 = 24/6

24/6 = 4

n/4 = 4

Multiply 4 by both sides to simplify:

n=16

4. n/56 = 10/16

10/16 = 0.625

n/56 = 0.625

Multiply 56 on both sides to simplify:

n=35

Step-by-step explanation:

how does the distributive property work

Answers

Explanation:

The distributive property tells you that multiplication distributes over addition. In equation form, it looks like ...

  a(b +c) = ab +ac

__

Groups and Multiples of Groups

The parentheses are a grouping symbol. In the above equation, they indicate that the sum (b+c) is to be treated as a group. When it is multiplied by 'a', everything in the group is multiplied by 'a'.

In the real world, there are many ways things are grouped. Baseball cards are packaged together with bubble gum; Cracker Jacks® are packaged together with a prize; left gloves are packaged together with right gloves; a bicycle is a package that has a frame, 2 wheels, a seat, handlebars. If you purchase a number of any of these packages, each of the items in the package is part of your purchase that number of times.

For example, 3 bicycles will have 3 frames, 3×2 = 6 wheels, 3 seats, 3 sets of handlebars. In algebraic form, this might look like ...

  3(f +2w + s + h) = 3f +6w +3s +3h

__

Eliminating Parentheses

Using the distributive property to eliminate parentheses is like regrouping the contents of the packages so like items are grouped together. In our above example, the 3 frames were grouped together when we considered that part of the group of 3 bicycles. The multiplier multiplies every member of the group.

__

Adding Parentheses

The reverse of the distributive property tells us we can regroup items into sets. It tells us we can take 3 frames, 6 wheels, 3 seats, and 3 handlebars and group them together in a way that makes 3 bicycles, each having a frame, 2 wheels, a seat, and handlebars.

  3f +6w +3s +3h = 3(f +2w + s + h)

Often, we're asked to choose the multiplier to be the greatest common factor (GCF) of the numbers of things in the group. That makes each group as small as possible. The multiplier only needs to be a common factor, not necessarily the greatest common factor.

What is the slope-intercept equation for this line?

y = [?]x+ [?]

Answers

Answer:

y=-x-3

Step-by-step explanation:

Graph the solution for the following linear inequality system. Click on the graph until the final result is displayed.
y ≥ 0
y < x
x + y < 6

Answers

Answer:

Step-by-step explanation:

(Score for Question 1:
of 5 points)
1. The diagram shown is two intersecting lines. The measure of 5 is 47°

(a) What is the measure of 7? How do you know. Explain your answer in complete sentences.
(b) Suppose the measure of 6 can be represented by (2x-5). What equation can be written to solve
for the value of x?
(c) What is the value of x?
Answer:

Answers

Answer:

a ground fault circuit interrupted

Answer:

By using what we know about intersecting lines, we will see that: a) ∡7 = 47° b) 47° + (2x - 5)° = 180° c) x = 69 How to get the measures of the angles for two intersecting lines? When two lines intersect, 4 angles are formed. There are two properties that these angles have. If the angles are adjacent, then their measures add up to 180° If the angles are not adjacent, these angles have the same measure. a) We know that ∡5 = 47° And 7 is not adjacent to 5, then 7 has the same measure: ∡7 = 47° b) 6 and 5 are adjacent, then we must have that: ∡5 + ∡6 = 180° 47° + (2x - 5)° = 180° That is the equation we need to solve to get the value of x. c) To solve the linear equation we need to isolate x, let's do that: 47° + (2x - 5)° = 180° (2x - 5)° = 180° - 47° = 133° 2x - 5 = 133 2x = 133 + 5 = 138 x = 138/2 = 69. If you want to learn more about linear equations, you can read: brainly.com/question/1884491

Find the value of each variable in the parallelogram.

Answers

Answer:

n=12

m=5

Step-by-step explanation:

n= 12 since they are parallel and equal

m+1=6 since they are parallel and equal

Opposite sides are equal in a paralleogram

m+1=6m=6-1m=5

And

n=12

Done!

I WILL MARK YOU THE BRAINLIEST LINKS WILL BE REPORTED AND DELETED !
What is the value of 7p7 ?
A: 1
B: 14
C: 49
D: 5040

Answers

Answer: 5040

Step-by-step explanation:

7•6•5•4•3•2•1=5040

Find the value of x x, y y, and z z in the parallelogram below.

Answers

In a parallelogram, angles that are opposite each other (as in, in opposite corners) are congruent. Angles that are next to each other (as in, on the same side) are supplementary.

It is very easy to solve for the value of y first because the expression is across from a known angle. Because these angles are opposite each other, they are congruent.

83 = 5y - 7

90 = 5y

y = 18

We can also use the same 83 degree angle to solve for the other two variables. 83 is on the same side as both of the remaining expressions, separately. As such 83 and another angle on the same side will add up to 180.

83 + -5x - 3 = 180

80 - 5x = 180

-5x = 100

x = -20

83 + z - 7 = 180

90 + z = 180

z = 90

Hope this helps!

3/5 + 1/4 = ?
Anyone know what the answer is?

Answers

17/20


you want to get the bottom numbers the same by multiplying:)
5x4= 20 , 4x5= 20
since you multiplied the fraction with the denominator 5 by 4 , you also multiply the number on top by 4
3x4= 12
and since the other fraction with the denominator 4 was multiplied by 5 , you multiply the number on top by 5
1x5= 5
now you add those 2 numbers up
5+12= 17
you put that number on top of the fraction: 17
and take the number you ————
got from the first step and
put it on the bottom: 20

and you get 17/20 or .85

This week you practice your instrument for 3 1/4 on Monday, 2 1/2 hours on Tuesday and 1 hour on Wednesday. Next week you practice for the same number of hours. What is the total number of hours you will have practiced over these six days?
Which expression correctly shows how to solve this problem?

A. 3 1/4 + 2 1/2 + 1
B. (3 1/4 + 2 1/2) + (3 1/4 + 2 1/2)
C. (3 1/4 + 2 1/2 + 1) + (3 1/4 + 2 1/2+ 1


Thank You so much!!!!

Answers

Answer:

C. (3 1/4 + 2 1/2 + 1) + (3 1/4 + 2 1/2+ 1)

Step-by-step explanation:

Since, this week you practice your instrument for 3 1/4 on Monday, 2 1/2 hours on Tuesday and 1 hour on Wednesday.

And, next week you practice for the same number of hours.

Same number = (3 1/4 + 2 1/2 + 1)

Thus, [C] (3 1/4 + 2 1/2 + 1) + (3 1/4 + 2 1/2+ 1) is correct.

~Lenvy~

Answer:

[C} [tex]\left(3\frac{1}{4}+2\frac{1}{2}+1\right)+\left(3\frac{1}{4}+2\frac{1}{2}+1\right)[/tex]

Step-by-step explanation:

Given:

this week you practice your instrument for 3 1/4 on Monday, 2 1/2 hours on Tuesday and 1 hour on Wednesday.

Next week you practice for the same number of hours.

To Find:

What is the total number of hours you will have practiced over these six days?

Which expression correctly shows how to solve this problem

Solve:

Knowing that you practice for 3 1/4 on Monday, 2 1/2 hours on Tuesday and 1 hour on Wednesday. Next week you practice the same. Thus, we can know that [C} [tex]\left(3\frac{1}{4}+2\frac{1}{2}+1\right)+\left(3\frac{1}{4}+2\frac{1}{2}+1\right)[/tex] is the correct answer.

Solution if needed.

[tex]\left(3\frac{1}{4}+2\frac{1}{2}+1\right)+\left(3\frac{1}{4}+2\frac{1}{2}+1\right)[/tex]

[tex]=\left(\frac{13}{4}+\frac{5}{2}+1\right)+\left(3\frac{1}{4}+2\frac{1}{2}+1\right)[/tex]

= [tex]\left(\frac{13}{4}+\frac{5}{2}+1\right)+\left(\frac{13}{4}+2\frac{1}{2}+1\right)[/tex]

[tex]=\left(\frac{13}{4}+\frac{5}{2}+1\right)+\left(\frac{13}{4}+\frac{5}{2}+1\right)[/tex]

[tex]\mathrm{Follow\:the\:PEMDAS\:order\:of\:operations}[/tex]

[tex]=\frac{27}{4}+\left(\frac{13}{4}+\frac{5}{2}+1\right)[/tex]

[tex]=\frac{27}{4}+\frac{27}{4}[/tex]

[tex]=\frac{27}{2}[/tex]

[tex]=13\frac{1}{2}[/tex]

Solution ⇒ [tex]=13\frac{1}{2}[/tex]

[RevyBreeze]

Other Questions
What factors led to the attacks on 9/11/01 by Islamic Fundamentalists? Please answer these true or false1: Cada noche durante la fiesta de Hanuka, las familias encienden un fuego artificial en la menora.2: Un aguinaldo es un regalo de Navidad.3: Los aquinaldos estan encima del rbol de Navidad.4: Los nios ponen decoraciones en el mausoleo de Navidad. Please I need help with this I have 5 questions that need help to answer 1) What are covalent bonds and how do they form ? 2) How do you know which elements and how many of each are in a compound ? 3) How can you find how many valence electrons an atom has ? 4) How many valence electrons do atoms need to be happy? Which elements are the exceptions? She says that the coffee cup fountain actually holds about 400 gallons of coffee.The newscaster claims that if the coffee fountain really broke, the 400 gallons of coffee would cover theequivalent of half a city block.You look up some information. One gallon of water covers about 13.8 square feet. Near the mall, an average city block is about 300 feet by 300 feet.Assume the fountain holds 400 gallons of coffee. Using the information given, determine whether thenewscaster's claim is correct. Construct an argument that supports or refutes the newscaster's claim.Include mathematics to support your argument. * PLEASE HELP ME SOLVE THIS PROBLEM I WILL GIVE BRAINIEST*2. Change from dog/cat to tails/heads.DDC = ________CDC= _________DCD = ________CCD = _________DDD = _________CCC = _________DCC = _________CDD = _________please help :)) What phrase does Jack's group use in Chapters 13 and 14 of Invisible Man that initially irritates the narrator? Read the following text from a student essay. How can the writer best improve his or her word choice?If I could invent something, I would create a really nice bicycle. My bike would have a good seat and great tires so that I could ride it anywhere. It would also have a bell and special pockets for carrying things. A special top would be wonderful, so I could ride it in the rain but also enjoy the sun. I would choose a fun color to reflect my personality. Inventing a new bike would be cool. Please give me the answers How to prepare for the ap calculus advanced placement examination Suppose you have a 100 mhz cpu and a a/d converter with an 8 khz sampling rate. Samples of the a/d converter are 32-bits A 5.00 L flask at 25 C contains 0.200 mol of Cl2 . What is the pressure in the flask? What does foil stand for in multiplying binomials brainly The bearing of a point K from a point L is 084 degrees. What is the bearing of L from K While the hidden curriculum certainly has it flaws, what are its benefits?A. It forwards the agenda of successful people.B. It plays an essential role in building social skills.C. It rarely puts forth negative messages about society.D. It is the only institution that helps shape morals and values. During WWII, the US and USSR were fighting against a common enemy (Germany & Fascism). When the war ended the common enemy and reason for cooperation disappeared. Why at that time did the US and USSR become involved in the Cold War?Question 7 options:The US & USSR realized a war would fuel their economiesThe US & USSR realized they had a solid disagreement in terms of basic beliefs on how to run a country (Communism versus Democracy)The US & USSR realized had a dispute about how to handle war debt The US & USSR realized they had several common values arr 1. Write a letter to the rector of The Polytechnic Ibadan in behalf of your class stating the problems facing the students at the department and suggest possible panacea to the problem so as to achieve the vision and mission statement of the department. In a market research survey of 2518 motorists, 252 said that they made an obscene gesture in the previous month.If one of the surveyed motorists is randomly selected, what is the probability that this motorist did not make an obscene gesture in the previous month? List the 3 factors that led to the decline of the Roman Empire What does this passage tell us most about Clover?She is anxious.She is timid.She is protective.She likes ducklings.